Difference between revisions of "2021 Fall AMC 12B Problems/Problem 10"

(Solution 1 (Quick Look for Symmetry))
(Solution 1 (Quick Look for Symmetry))
Line 13: Line 13:
  
 
Thus we have <math>20+50+80+230=\boxed{(\textbf{E})\ 380}</math>.
 
Thus we have <math>20+50+80+230=\boxed{(\textbf{E})\ 380}</math>.
 +
 +
Note: You may check this with a diagram featuring a unit circle and the above angles for polar coordinates.
  
 
~Wilhelm Z
 
~Wilhelm Z

Revision as of 04:02, 25 November 2021

Problem

What is the sum of all possible values of $t$ between $0$ and $360$ such that the triangle in the coordinate plane whose vertices are $(\cos(40^\circ),\sin(40^\circ))$, $(\cos(60^\circ),\sin(60^\circ))$, and $(\cos(t^\circ),\sin(t^\circ))$ is isosceles?

$\textbf{(A)} \: 100 \qquad\textbf{(B)} \: 150 \qquad\textbf{(C)} \: 330 \qquad\textbf{(D)} \: 360 \qquad\textbf{(E)} \: 380$

Solution 1 (Quick Look for Symmetry)

By inspection, we may obtain the following choices for which symmetric isosceles triangles could be constructed within the unit circle described:

$20^\circ$, $50^\circ$, $80^\circ$, and $230^\circ$.

Thus we have $20+50+80+230=\boxed{(\textbf{E})\ 380}$.

Note: You may check this with a diagram featuring a unit circle and the above angles for polar coordinates.

~Wilhelm Z

2021 Fall AMC 12B (ProblemsAnswer KeyResources)
Preceded by
Problem 9
Followed by
Problem 11
1 2 3 4 5 6 7 8 9 10 11 12 13 14 15 16 17 18 19 20 21 22 23 24 25
All AMC 12 Problems and Solutions

The problems on this page are copyrighted by the Mathematical Association of America's American Mathematics Competitions. AMC logo.png